Solving for Angle in Projectile Motion: A Step-by-Step Guide

  • Thread starter blackboy
  • Start date
  • Tags
    Motion
In summary, the problem is to determine the angle at which a cannon should be fired at a mountain slope to start an avalanche. Using kinematic equations and trigonometric identities, the angle is found to be approximately 22.36 degrees or 89.43 degrees.
  • #1
blackboy
61
0

Homework Statement


A cannon with muzzle speed of 1000 m/s is used to start an avalanche on a mountain slope. The target 2000m from the cannon horizontally and 800m above the cannon. At what angle should the cannon be fired?

Homework Equations


A buttload of the projectile motion ones.

The Attempt at a Solution


Ok what I did was I tried to solve for t and get equations for the angle, but my lack of trig gets in the way. Ok for no better thing to use let :smile:=theta. I get 1000cos(:smile:)t=2000 so cos(:smile:)t=2. Then for the y component I get 800=sin(:smile:)t-4.9t^2. I solved here for t with the quadratic equation. Substituting the resulting gives me 19.6=500sin(2:smile:)+64sqrt[15625sin^2(:smile:)-245]. I don't know how to solve for :smile:. Help please! Thank you!
 
Physics news on Phys.org
  • #2
blackboy said:
Then for the y component I get 800=sin(:smile:)t-4.9t^2.

Should be 800 = 1000sin(:smile:)t - 4.9t2.
 
  • #3
Yeah I just forgot to type that there, however I do not know how to solve the trig equation there.
 
  • #4
Using your kinematic equations, you know...

[tex]\begin{array}{l}
y = \sin (\theta )v_0 t + \frac{{gt^2 }}{2} \\
x = \cos (\theta )v_0 t \\
\end{array}[/tex]

If you isolate the trigonometric parts on the right side, you can divide the equations and find [tex]\frac{{y - \frac{{gt^2 }}{2}}}{x} = \frac{{\sin (\theta )v_0 }}{{\cos (\theta )v_0 }}[/tex]

Now the problem is that t is still in there. I would suggest substituting [tex]t = \frac{x}{{\cos (\theta )v_0 }}[/tex]. Do some manipulation and you can determine your angle! If you need to use trigonometric identities, i highly suggest checking out http://en.wikipedia.org/wiki/Trigonometric_identities and really getting down the use of the identities.

Note, there are 2 solutions to this problem. This is simply because there is a very large angle that you can shoot the cannon upwards such that it'll hit its target after its long but narrow arc is completed and it's falling downward.
 
Last edited:
  • #5
Suppressing the constants, you should get an equation like

[tex] \frac{\sin \theta}{\cos \theta} + \frac{1}{\cos^2 \theta} = 1 [/tex]

Wrtie sin(θ)/cos(θ) as tan(θ), and 1/cos2(θ) as 1 + tan2(θ).

Putting constants back, you will get a quadratic in tan like tan2(θ) + B tan(θ) + C = 0.
 
Last edited:
  • #6
Solving for :smile: we get tan(:smile:)=0,-1. That is an angle of 0 degrees or 135 or 225? What?

Oops sorry forgot about the constants..
 
Last edited:
  • #7
What equation did you solve? This is the one you should have gotten:

819.6 = 2000 tan(θ) - 19.6 tan2(θ)
 
  • #8
Um can you show me where you got those constants from?
 
  • #9
You had the equation

800 = 1000sin(θ)T - 4.9T2

Now just substitue T = 2/cos(θ) in this.
 
  • #10
Ok I learned that 1/cos^2(:smile:)= Tan^2(:smile:)+1. That helped a lot so solving for :smile:, I finally get :smile:=22.36 or 89.43. Thanks a bunch both of you! BTW when did you guys learn these trig identities? I did not see that 1/cos^2(:smile:)=tan^2(:smile:)+1 on the wikipedia page.
 
  • #11
I didn't see it at first either. I was breaking my head trying to solve it for cos(θ)! I am surprised 1/cos^2(θ)=tan^2(θ)+1 is not listed on wiki, it's a pretty standard identity, usually written in the form sec^2(θ) = 1 + tan^2(θ).
 

1. What is projectile motion and how does it relate to a cannon?

Projectile motion refers to the curved path that an object takes when it is launched into the air and is subject to the force of gravity. This motion is often seen in the trajectory of a cannonball when it is fired from a cannon.

2. What factors affect the projectile motion of a cannonball?

The factors that affect the projectile motion of a cannonball include the initial velocity, launch angle, air resistance, and the force of gravity. These factors can influence the height, distance, and time of flight of the cannonball.

3. How does the angle of elevation affect the range of a cannonball?

The angle of elevation, or the angle at which the cannon is fired, plays a crucial role in determining the range of a cannonball. The optimal angle of elevation for maximum range is 45 degrees, as this angle allows for the most efficient use of the initial velocity and gravitational force.

4. Can air resistance be ignored in projectile motion calculations for a cannon?

No, air resistance cannot be ignored in projectile motion calculations for a cannon. While it may be negligible for small objects, the air resistance on a large, heavy object like a cannonball can significantly affect its trajectory and should be taken into account in calculations.

5. What is the maximum height reached by a cannonball during projectile motion?

The maximum height reached by a cannonball during projectile motion depends on the initial velocity and launch angle. The formula for calculating this height is h = (v^2 * sin^2θ)/(2g), where v is the initial velocity, θ is the launch angle, and g is the acceleration due to gravity. So, the maximum height can vary depending on these factors.

Similar threads

  • Introductory Physics Homework Help
Replies
11
Views
798
  • Introductory Physics Homework Help
Replies
7
Views
2K
Replies
2
Views
2K
  • Introductory Physics Homework Help
Replies
15
Views
1K
  • Introductory Physics Homework Help
Replies
1
Views
1K
  • Introductory Physics Homework Help
Replies
19
Views
1K
  • Introductory Physics Homework Help
Replies
5
Views
1K
  • Introductory Physics Homework Help
Replies
4
Views
862
  • Introductory Physics Homework Help
Replies
6
Views
1K
  • Introductory Physics Homework Help
Replies
6
Views
201
Back
Top